Evaluate $int_C frac{e^{1/z}}{(z-i)^3}dz$ in the circle $|z|=5$












2















Evaluate $int_C frac{e^{1/z}}{(z-i)^3}dz$ in the circle $|z|=5$




I want to check that if my solution is correct.



Since $1/z$ is not analytic at $z=0$ and $1/(z-i)^3$ is not analytic at $z=i$, hence the function $f(z) =frac{e^{1/z}}{(z-i)^3}$ has singularities at $z=0$ and $z=3$.



Notice that both of them are inside the circle $|z|=5$.



So we can use residue at infinity to evaluate this integral.



We have:



$f(1/z)= frac{e^z}{(1/z-i)^3} = frac{z^3e^z}{(1-iz)^3} implies frac{1}{z^2}f(1/z) = frac{ze^z}{(1-iz)^3}.$



We must find the residue of $frac{1}{z^2}f(1/z)$ at $z=0$. But notice that this function is analytic at this point. Hence its Laurent expansion is equal to its Taylor expansion. Its principal part is then zero, so we have



$mbox{Res}_{z = 0}frac{1}{z^2}f(1/z) = 0$.



So, finally: $int_C frac{e^{1/z}}{(z-i)^3}dz = 2pi i*0 = 0.$



Is this solution correct? I am asking because I did not need to find any series expansion!










share|cite|improve this question


















  • 1




    Yes, this is ok. (Taking the residue at $infty$ is related to the integral on the curve $C$ taken with converse orientation, so as a joke, the result is $-0$.) Alternatively, one can make a change of variables $w=1/z$, this brings the given integral in one over the circle $C'$ with radius $1/5$, with no singularity in $w=0$, and the singularity in $w=-i$ is outside the integral contour $C'$.
    – dan_fulea
    Nov 29 '18 at 18:39
















2















Evaluate $int_C frac{e^{1/z}}{(z-i)^3}dz$ in the circle $|z|=5$




I want to check that if my solution is correct.



Since $1/z$ is not analytic at $z=0$ and $1/(z-i)^3$ is not analytic at $z=i$, hence the function $f(z) =frac{e^{1/z}}{(z-i)^3}$ has singularities at $z=0$ and $z=3$.



Notice that both of them are inside the circle $|z|=5$.



So we can use residue at infinity to evaluate this integral.



We have:



$f(1/z)= frac{e^z}{(1/z-i)^3} = frac{z^3e^z}{(1-iz)^3} implies frac{1}{z^2}f(1/z) = frac{ze^z}{(1-iz)^3}.$



We must find the residue of $frac{1}{z^2}f(1/z)$ at $z=0$. But notice that this function is analytic at this point. Hence its Laurent expansion is equal to its Taylor expansion. Its principal part is then zero, so we have



$mbox{Res}_{z = 0}frac{1}{z^2}f(1/z) = 0$.



So, finally: $int_C frac{e^{1/z}}{(z-i)^3}dz = 2pi i*0 = 0.$



Is this solution correct? I am asking because I did not need to find any series expansion!










share|cite|improve this question


















  • 1




    Yes, this is ok. (Taking the residue at $infty$ is related to the integral on the curve $C$ taken with converse orientation, so as a joke, the result is $-0$.) Alternatively, one can make a change of variables $w=1/z$, this brings the given integral in one over the circle $C'$ with radius $1/5$, with no singularity in $w=0$, and the singularity in $w=-i$ is outside the integral contour $C'$.
    – dan_fulea
    Nov 29 '18 at 18:39














2












2








2








Evaluate $int_C frac{e^{1/z}}{(z-i)^3}dz$ in the circle $|z|=5$




I want to check that if my solution is correct.



Since $1/z$ is not analytic at $z=0$ and $1/(z-i)^3$ is not analytic at $z=i$, hence the function $f(z) =frac{e^{1/z}}{(z-i)^3}$ has singularities at $z=0$ and $z=3$.



Notice that both of them are inside the circle $|z|=5$.



So we can use residue at infinity to evaluate this integral.



We have:



$f(1/z)= frac{e^z}{(1/z-i)^3} = frac{z^3e^z}{(1-iz)^3} implies frac{1}{z^2}f(1/z) = frac{ze^z}{(1-iz)^3}.$



We must find the residue of $frac{1}{z^2}f(1/z)$ at $z=0$. But notice that this function is analytic at this point. Hence its Laurent expansion is equal to its Taylor expansion. Its principal part is then zero, so we have



$mbox{Res}_{z = 0}frac{1}{z^2}f(1/z) = 0$.



So, finally: $int_C frac{e^{1/z}}{(z-i)^3}dz = 2pi i*0 = 0.$



Is this solution correct? I am asking because I did not need to find any series expansion!










share|cite|improve this question














Evaluate $int_C frac{e^{1/z}}{(z-i)^3}dz$ in the circle $|z|=5$




I want to check that if my solution is correct.



Since $1/z$ is not analytic at $z=0$ and $1/(z-i)^3$ is not analytic at $z=i$, hence the function $f(z) =frac{e^{1/z}}{(z-i)^3}$ has singularities at $z=0$ and $z=3$.



Notice that both of them are inside the circle $|z|=5$.



So we can use residue at infinity to evaluate this integral.



We have:



$f(1/z)= frac{e^z}{(1/z-i)^3} = frac{z^3e^z}{(1-iz)^3} implies frac{1}{z^2}f(1/z) = frac{ze^z}{(1-iz)^3}.$



We must find the residue of $frac{1}{z^2}f(1/z)$ at $z=0$. But notice that this function is analytic at this point. Hence its Laurent expansion is equal to its Taylor expansion. Its principal part is then zero, so we have



$mbox{Res}_{z = 0}frac{1}{z^2}f(1/z) = 0$.



So, finally: $int_C frac{e^{1/z}}{(z-i)^3}dz = 2pi i*0 = 0.$



Is this solution correct? I am asking because I did not need to find any series expansion!







complex-analysis proof-verification






share|cite|improve this question













share|cite|improve this question











share|cite|improve this question




share|cite|improve this question










asked Nov 29 '18 at 18:13









math.h

1,122517




1,122517








  • 1




    Yes, this is ok. (Taking the residue at $infty$ is related to the integral on the curve $C$ taken with converse orientation, so as a joke, the result is $-0$.) Alternatively, one can make a change of variables $w=1/z$, this brings the given integral in one over the circle $C'$ with radius $1/5$, with no singularity in $w=0$, and the singularity in $w=-i$ is outside the integral contour $C'$.
    – dan_fulea
    Nov 29 '18 at 18:39














  • 1




    Yes, this is ok. (Taking the residue at $infty$ is related to the integral on the curve $C$ taken with converse orientation, so as a joke, the result is $-0$.) Alternatively, one can make a change of variables $w=1/z$, this brings the given integral in one over the circle $C'$ with radius $1/5$, with no singularity in $w=0$, and the singularity in $w=-i$ is outside the integral contour $C'$.
    – dan_fulea
    Nov 29 '18 at 18:39








1




1




Yes, this is ok. (Taking the residue at $infty$ is related to the integral on the curve $C$ taken with converse orientation, so as a joke, the result is $-0$.) Alternatively, one can make a change of variables $w=1/z$, this brings the given integral in one over the circle $C'$ with radius $1/5$, with no singularity in $w=0$, and the singularity in $w=-i$ is outside the integral contour $C'$.
– dan_fulea
Nov 29 '18 at 18:39




Yes, this is ok. (Taking the residue at $infty$ is related to the integral on the curve $C$ taken with converse orientation, so as a joke, the result is $-0$.) Alternatively, one can make a change of variables $w=1/z$, this brings the given integral in one over the circle $C'$ with radius $1/5$, with no singularity in $w=0$, and the singularity in $w=-i$ is outside the integral contour $C'$.
– dan_fulea
Nov 29 '18 at 18:39










4 Answers
4






active

oldest

votes


















1














Yes. Or use the following more general argument. Whenever the integrand is regular outside the contour, we can make the contour progressively larger (radius $R$) without altering the integral, by Cauchy's thm, and if the integrand is $o(1/R)$ on the contour we will always get zero, as by the Bounding Lemma the integral is less than circumference of contour $times$ maximum of integrand, which goes as $2pi R times o(1/R) to 0$. In effect this amounts to changing variable $zmapsto 1/z$ and using analyticity.






share|cite|improve this answer



















  • 2




    The contour is fixed from the start.
    – José Carlos Santos
    Nov 29 '18 at 18:23










  • Well, how about using Cauchy's theorem.
    – Richard Martin
    Nov 29 '18 at 18:26






  • 1




    Use it then. I will not try to stop you.
    – José Carlos Santos
    Nov 29 '18 at 18:27










  • Thought this was obvious but perhaps not ... see edits.
    – Richard Martin
    Nov 30 '18 at 11:34










  • @RichardMartin This is the approach that I often prefer over evaluating the residue at infinity. I've posted a solution that illustrates the approach for the integral of interest herein. (+1) and Happy Holidays!
    – Mark Viola
    Nov 30 '18 at 18:07



















1














We have, $displaystyle fleft(frac 1zright)=frac{z^3e^z}{(1-iz)^3}$.



Now, $displaystyle Res(f(z),infty)=Resleft(frac{1}{z^2}f(1/z),0right)=Resleft(frac{ze^z}{(1-iz)^3},0right)=0 text{ as it is analytic at $z=0$.}$



Now, Sum of the residues at the finite poles and the residue at infinity is $0$.



So, $Res(f,0)+Res(f,i)=-Res(f,infty)=0.$



Therefore by Cauchy's residue Theorem integral value is $0$.






share|cite|improve this answer































    0














    OK, i will type an answer, after the comment. The result $0$ is correct. We change variables, $w=1/z$. The contour $C$, circle with radius $5$ goes into the circle $C'$ with radius $1/5$. Then we have
    $$
    begin{aligned}
    int_Cfrac {e^{1/z}}{(z-i)^3}; dz
    &=
    int_{C'}frac{e^w}{left(frac 1w-iright)^3}; left(-frac 1{w^2}right);dw
    \
    &=
    -int_{C'}frac {w^3e^w}{w^2(1-iw)^3};dw
    \
    &=
    -int_{C'}frac {we^w}{(1-iw)^3};dw
    .
    end{aligned}
    $$

    There is no pole inside $C'$. (The pole $w=-i$ is outside.)
    So the integral is zero, Residue Theorem.



    Numerically, pari/gp:



    ? intnum( t=0, 2*Pi, 
    exp( (cos(t)-I*sin(t))/5 )
    / (5*(cos(t)+I*sin(t)) - I)^3
    * 5*(-sin(t)+I*cos(t)) )
    %1 = 1.4190890450537301016459116208947201222 E-21
    - 1.9571617134379963221261930053722380946 E-21*I


    (Manually broken lines to fit in the window.)





    This is of course better than computing the two residues of the initial function (in $0$ and in $i$), which are in $i$...



    sage: var('z');
    sage: f = exp(1/z) / (z-i)^3
    sage: f.residue(z==i)
    (I + 1/2)*e^(-I)


    computed with sage, and in $0$ with the hand
    $$
    begin{aligned}
    &text{Coeff}_{1/z}
    frac {e^{1/z}}{(z-i)^3}
    \
    &=
    text{Coeff}_{1/z}
    -icdot e^{1/z}cdotfrac 1{(1+iz)^3}
    \
    &=text{Coeff}_{1/z}
    -icdot
    left(
    frac 1{0!}
    +frac 1{1!}frac 1z
    +frac 1{2!}frac 1{z^2}
    +frac 1{3!}frac 1{z^3}
    +dotsright)
    \
    &qquadcdot
    left(
    binom 22
    +binom 32 (-iz)
    +binom 42(-iz)^2
    %+binom 52(-iz)^3
    +dots
    right)
    \
    &=
    -ileft(
    frac 1{1!}binom 22
    +frac 1{2!}binom 32 (-i)
    +frac 1{3!}binom 42 (-i)^2
    %+frac 1{4!}binom 52 (-i)^3
    +dots
    right)
    \
    &=dots
    end{aligned}
    $$

    and in the above sum each $frac 1{(n+1)!}binom{n+2}2
    =frac 1{(n+1)!}cdotfrac{(n+2)!}{n!2!}=frac{(n+2)}{n!2!}$
    has to be explained. We split the nummerator in two parts, one with $n$, one with $2$, and will get some parts of the exponential series in $-i$. One of them needs the aid of the missing $1/0!$, so it is acceptable that we get the killing contribution for the first residue.






    share|cite|improve this answer





























      0















      I thought it might be instructive to present an approach that is equivalent to using the residue at infinity, but perhaps a bit simpler in this case. To that end we proceed.






      Inasmuch as the singularities of the integrand lie inside the unit circle, Cauchy's Integral Theorem guarantees that for $R>5$



      $$oint_{|z|=5}frac{e^{1/z}}{(z-i)^3},dz=oint_{|z|=R}frac{e^{1/z}}{(z-i)^3},dz$$



      Now, we have the simple estimate



      $$begin{align}
      left|oint_{|z|=R}frac{e^{1/z}}{(z-i)^3},dzright|&=left|int_0^{2pi} frac{e^{frac1Re^{-iphi}}}{(Re^{iphi}-i)^3},iRe^{iphi},dphiright|\\
      &le frac{2pi R^2}{(R-1)^4}
      end{align}$$



      Finally, letting $Rto infty$, we find



      $$oint_{|z|=5}frac{e^{1/z}}{(z-i)^3},dz=0$$



      And we are done!






      share|cite|improve this answer





















      • Please let me know how I can improve my answer. I really want to give you the best answer I can. And Happy Holidays! ;-)
        – Mark Viola
        Dec 16 '18 at 18:23











      Your Answer





      StackExchange.ifUsing("editor", function () {
      return StackExchange.using("mathjaxEditing", function () {
      StackExchange.MarkdownEditor.creationCallbacks.add(function (editor, postfix) {
      StackExchange.mathjaxEditing.prepareWmdForMathJax(editor, postfix, [["$", "$"], ["\\(","\\)"]]);
      });
      });
      }, "mathjax-editing");

      StackExchange.ready(function() {
      var channelOptions = {
      tags: "".split(" "),
      id: "69"
      };
      initTagRenderer("".split(" "), "".split(" "), channelOptions);

      StackExchange.using("externalEditor", function() {
      // Have to fire editor after snippets, if snippets enabled
      if (StackExchange.settings.snippets.snippetsEnabled) {
      StackExchange.using("snippets", function() {
      createEditor();
      });
      }
      else {
      createEditor();
      }
      });

      function createEditor() {
      StackExchange.prepareEditor({
      heartbeatType: 'answer',
      autoActivateHeartbeat: false,
      convertImagesToLinks: true,
      noModals: true,
      showLowRepImageUploadWarning: true,
      reputationToPostImages: 10,
      bindNavPrevention: true,
      postfix: "",
      imageUploader: {
      brandingHtml: "Powered by u003ca class="icon-imgur-white" href="https://imgur.com/"u003eu003c/au003e",
      contentPolicyHtml: "User contributions licensed under u003ca href="https://creativecommons.org/licenses/by-sa/3.0/"u003ecc by-sa 3.0 with attribution requiredu003c/au003e u003ca href="https://stackoverflow.com/legal/content-policy"u003e(content policy)u003c/au003e",
      allowUrls: true
      },
      noCode: true, onDemand: true,
      discardSelector: ".discard-answer"
      ,immediatelyShowMarkdownHelp:true
      });


      }
      });














      draft saved

      draft discarded


















      StackExchange.ready(
      function () {
      StackExchange.openid.initPostLogin('.new-post-login', 'https%3a%2f%2fmath.stackexchange.com%2fquestions%2f3018976%2fevaluate-int-c-frace1-zz-i3dz-in-the-circle-z-5%23new-answer', 'question_page');
      }
      );

      Post as a guest















      Required, but never shown

























      4 Answers
      4






      active

      oldest

      votes








      4 Answers
      4






      active

      oldest

      votes









      active

      oldest

      votes






      active

      oldest

      votes









      1














      Yes. Or use the following more general argument. Whenever the integrand is regular outside the contour, we can make the contour progressively larger (radius $R$) without altering the integral, by Cauchy's thm, and if the integrand is $o(1/R)$ on the contour we will always get zero, as by the Bounding Lemma the integral is less than circumference of contour $times$ maximum of integrand, which goes as $2pi R times o(1/R) to 0$. In effect this amounts to changing variable $zmapsto 1/z$ and using analyticity.






      share|cite|improve this answer



















      • 2




        The contour is fixed from the start.
        – José Carlos Santos
        Nov 29 '18 at 18:23










      • Well, how about using Cauchy's theorem.
        – Richard Martin
        Nov 29 '18 at 18:26






      • 1




        Use it then. I will not try to stop you.
        – José Carlos Santos
        Nov 29 '18 at 18:27










      • Thought this was obvious but perhaps not ... see edits.
        – Richard Martin
        Nov 30 '18 at 11:34










      • @RichardMartin This is the approach that I often prefer over evaluating the residue at infinity. I've posted a solution that illustrates the approach for the integral of interest herein. (+1) and Happy Holidays!
        – Mark Viola
        Nov 30 '18 at 18:07
















      1














      Yes. Or use the following more general argument. Whenever the integrand is regular outside the contour, we can make the contour progressively larger (radius $R$) without altering the integral, by Cauchy's thm, and if the integrand is $o(1/R)$ on the contour we will always get zero, as by the Bounding Lemma the integral is less than circumference of contour $times$ maximum of integrand, which goes as $2pi R times o(1/R) to 0$. In effect this amounts to changing variable $zmapsto 1/z$ and using analyticity.






      share|cite|improve this answer



















      • 2




        The contour is fixed from the start.
        – José Carlos Santos
        Nov 29 '18 at 18:23










      • Well, how about using Cauchy's theorem.
        – Richard Martin
        Nov 29 '18 at 18:26






      • 1




        Use it then. I will not try to stop you.
        – José Carlos Santos
        Nov 29 '18 at 18:27










      • Thought this was obvious but perhaps not ... see edits.
        – Richard Martin
        Nov 30 '18 at 11:34










      • @RichardMartin This is the approach that I often prefer over evaluating the residue at infinity. I've posted a solution that illustrates the approach for the integral of interest herein. (+1) and Happy Holidays!
        – Mark Viola
        Nov 30 '18 at 18:07














      1












      1








      1






      Yes. Or use the following more general argument. Whenever the integrand is regular outside the contour, we can make the contour progressively larger (radius $R$) without altering the integral, by Cauchy's thm, and if the integrand is $o(1/R)$ on the contour we will always get zero, as by the Bounding Lemma the integral is less than circumference of contour $times$ maximum of integrand, which goes as $2pi R times o(1/R) to 0$. In effect this amounts to changing variable $zmapsto 1/z$ and using analyticity.






      share|cite|improve this answer














      Yes. Or use the following more general argument. Whenever the integrand is regular outside the contour, we can make the contour progressively larger (radius $R$) without altering the integral, by Cauchy's thm, and if the integrand is $o(1/R)$ on the contour we will always get zero, as by the Bounding Lemma the integral is less than circumference of contour $times$ maximum of integrand, which goes as $2pi R times o(1/R) to 0$. In effect this amounts to changing variable $zmapsto 1/z$ and using analyticity.







      share|cite|improve this answer














      share|cite|improve this answer



      share|cite|improve this answer








      edited Nov 30 '18 at 11:32

























      answered Nov 29 '18 at 18:21









      Richard Martin

      1,61118




      1,61118








      • 2




        The contour is fixed from the start.
        – José Carlos Santos
        Nov 29 '18 at 18:23










      • Well, how about using Cauchy's theorem.
        – Richard Martin
        Nov 29 '18 at 18:26






      • 1




        Use it then. I will not try to stop you.
        – José Carlos Santos
        Nov 29 '18 at 18:27










      • Thought this was obvious but perhaps not ... see edits.
        – Richard Martin
        Nov 30 '18 at 11:34










      • @RichardMartin This is the approach that I often prefer over evaluating the residue at infinity. I've posted a solution that illustrates the approach for the integral of interest herein. (+1) and Happy Holidays!
        – Mark Viola
        Nov 30 '18 at 18:07














      • 2




        The contour is fixed from the start.
        – José Carlos Santos
        Nov 29 '18 at 18:23










      • Well, how about using Cauchy's theorem.
        – Richard Martin
        Nov 29 '18 at 18:26






      • 1




        Use it then. I will not try to stop you.
        – José Carlos Santos
        Nov 29 '18 at 18:27










      • Thought this was obvious but perhaps not ... see edits.
        – Richard Martin
        Nov 30 '18 at 11:34










      • @RichardMartin This is the approach that I often prefer over evaluating the residue at infinity. I've posted a solution that illustrates the approach for the integral of interest herein. (+1) and Happy Holidays!
        – Mark Viola
        Nov 30 '18 at 18:07








      2




      2




      The contour is fixed from the start.
      – José Carlos Santos
      Nov 29 '18 at 18:23




      The contour is fixed from the start.
      – José Carlos Santos
      Nov 29 '18 at 18:23












      Well, how about using Cauchy's theorem.
      – Richard Martin
      Nov 29 '18 at 18:26




      Well, how about using Cauchy's theorem.
      – Richard Martin
      Nov 29 '18 at 18:26




      1




      1




      Use it then. I will not try to stop you.
      – José Carlos Santos
      Nov 29 '18 at 18:27




      Use it then. I will not try to stop you.
      – José Carlos Santos
      Nov 29 '18 at 18:27












      Thought this was obvious but perhaps not ... see edits.
      – Richard Martin
      Nov 30 '18 at 11:34




      Thought this was obvious but perhaps not ... see edits.
      – Richard Martin
      Nov 30 '18 at 11:34












      @RichardMartin This is the approach that I often prefer over evaluating the residue at infinity. I've posted a solution that illustrates the approach for the integral of interest herein. (+1) and Happy Holidays!
      – Mark Viola
      Nov 30 '18 at 18:07




      @RichardMartin This is the approach that I often prefer over evaluating the residue at infinity. I've posted a solution that illustrates the approach for the integral of interest herein. (+1) and Happy Holidays!
      – Mark Viola
      Nov 30 '18 at 18:07











      1














      We have, $displaystyle fleft(frac 1zright)=frac{z^3e^z}{(1-iz)^3}$.



      Now, $displaystyle Res(f(z),infty)=Resleft(frac{1}{z^2}f(1/z),0right)=Resleft(frac{ze^z}{(1-iz)^3},0right)=0 text{ as it is analytic at $z=0$.}$



      Now, Sum of the residues at the finite poles and the residue at infinity is $0$.



      So, $Res(f,0)+Res(f,i)=-Res(f,infty)=0.$



      Therefore by Cauchy's residue Theorem integral value is $0$.






      share|cite|improve this answer




























        1














        We have, $displaystyle fleft(frac 1zright)=frac{z^3e^z}{(1-iz)^3}$.



        Now, $displaystyle Res(f(z),infty)=Resleft(frac{1}{z^2}f(1/z),0right)=Resleft(frac{ze^z}{(1-iz)^3},0right)=0 text{ as it is analytic at $z=0$.}$



        Now, Sum of the residues at the finite poles and the residue at infinity is $0$.



        So, $Res(f,0)+Res(f,i)=-Res(f,infty)=0.$



        Therefore by Cauchy's residue Theorem integral value is $0$.






        share|cite|improve this answer


























          1












          1








          1






          We have, $displaystyle fleft(frac 1zright)=frac{z^3e^z}{(1-iz)^3}$.



          Now, $displaystyle Res(f(z),infty)=Resleft(frac{1}{z^2}f(1/z),0right)=Resleft(frac{ze^z}{(1-iz)^3},0right)=0 text{ as it is analytic at $z=0$.}$



          Now, Sum of the residues at the finite poles and the residue at infinity is $0$.



          So, $Res(f,0)+Res(f,i)=-Res(f,infty)=0.$



          Therefore by Cauchy's residue Theorem integral value is $0$.






          share|cite|improve this answer














          We have, $displaystyle fleft(frac 1zright)=frac{z^3e^z}{(1-iz)^3}$.



          Now, $displaystyle Res(f(z),infty)=Resleft(frac{1}{z^2}f(1/z),0right)=Resleft(frac{ze^z}{(1-iz)^3},0right)=0 text{ as it is analytic at $z=0$.}$



          Now, Sum of the residues at the finite poles and the residue at infinity is $0$.



          So, $Res(f,0)+Res(f,i)=-Res(f,infty)=0.$



          Therefore by Cauchy's residue Theorem integral value is $0$.







          share|cite|improve this answer














          share|cite|improve this answer



          share|cite|improve this answer








          edited Nov 30 '18 at 17:27

























          answered Nov 30 '18 at 17:21









          Empty

          8,07752560




          8,07752560























              0














              OK, i will type an answer, after the comment. The result $0$ is correct. We change variables, $w=1/z$. The contour $C$, circle with radius $5$ goes into the circle $C'$ with radius $1/5$. Then we have
              $$
              begin{aligned}
              int_Cfrac {e^{1/z}}{(z-i)^3}; dz
              &=
              int_{C'}frac{e^w}{left(frac 1w-iright)^3}; left(-frac 1{w^2}right);dw
              \
              &=
              -int_{C'}frac {w^3e^w}{w^2(1-iw)^3};dw
              \
              &=
              -int_{C'}frac {we^w}{(1-iw)^3};dw
              .
              end{aligned}
              $$

              There is no pole inside $C'$. (The pole $w=-i$ is outside.)
              So the integral is zero, Residue Theorem.



              Numerically, pari/gp:



              ? intnum( t=0, 2*Pi, 
              exp( (cos(t)-I*sin(t))/5 )
              / (5*(cos(t)+I*sin(t)) - I)^3
              * 5*(-sin(t)+I*cos(t)) )
              %1 = 1.4190890450537301016459116208947201222 E-21
              - 1.9571617134379963221261930053722380946 E-21*I


              (Manually broken lines to fit in the window.)





              This is of course better than computing the two residues of the initial function (in $0$ and in $i$), which are in $i$...



              sage: var('z');
              sage: f = exp(1/z) / (z-i)^3
              sage: f.residue(z==i)
              (I + 1/2)*e^(-I)


              computed with sage, and in $0$ with the hand
              $$
              begin{aligned}
              &text{Coeff}_{1/z}
              frac {e^{1/z}}{(z-i)^3}
              \
              &=
              text{Coeff}_{1/z}
              -icdot e^{1/z}cdotfrac 1{(1+iz)^3}
              \
              &=text{Coeff}_{1/z}
              -icdot
              left(
              frac 1{0!}
              +frac 1{1!}frac 1z
              +frac 1{2!}frac 1{z^2}
              +frac 1{3!}frac 1{z^3}
              +dotsright)
              \
              &qquadcdot
              left(
              binom 22
              +binom 32 (-iz)
              +binom 42(-iz)^2
              %+binom 52(-iz)^3
              +dots
              right)
              \
              &=
              -ileft(
              frac 1{1!}binom 22
              +frac 1{2!}binom 32 (-i)
              +frac 1{3!}binom 42 (-i)^2
              %+frac 1{4!}binom 52 (-i)^3
              +dots
              right)
              \
              &=dots
              end{aligned}
              $$

              and in the above sum each $frac 1{(n+1)!}binom{n+2}2
              =frac 1{(n+1)!}cdotfrac{(n+2)!}{n!2!}=frac{(n+2)}{n!2!}$
              has to be explained. We split the nummerator in two parts, one with $n$, one with $2$, and will get some parts of the exponential series in $-i$. One of them needs the aid of the missing $1/0!$, so it is acceptable that we get the killing contribution for the first residue.






              share|cite|improve this answer


























                0














                OK, i will type an answer, after the comment. The result $0$ is correct. We change variables, $w=1/z$. The contour $C$, circle with radius $5$ goes into the circle $C'$ with radius $1/5$. Then we have
                $$
                begin{aligned}
                int_Cfrac {e^{1/z}}{(z-i)^3}; dz
                &=
                int_{C'}frac{e^w}{left(frac 1w-iright)^3}; left(-frac 1{w^2}right);dw
                \
                &=
                -int_{C'}frac {w^3e^w}{w^2(1-iw)^3};dw
                \
                &=
                -int_{C'}frac {we^w}{(1-iw)^3};dw
                .
                end{aligned}
                $$

                There is no pole inside $C'$. (The pole $w=-i$ is outside.)
                So the integral is zero, Residue Theorem.



                Numerically, pari/gp:



                ? intnum( t=0, 2*Pi, 
                exp( (cos(t)-I*sin(t))/5 )
                / (5*(cos(t)+I*sin(t)) - I)^3
                * 5*(-sin(t)+I*cos(t)) )
                %1 = 1.4190890450537301016459116208947201222 E-21
                - 1.9571617134379963221261930053722380946 E-21*I


                (Manually broken lines to fit in the window.)





                This is of course better than computing the two residues of the initial function (in $0$ and in $i$), which are in $i$...



                sage: var('z');
                sage: f = exp(1/z) / (z-i)^3
                sage: f.residue(z==i)
                (I + 1/2)*e^(-I)


                computed with sage, and in $0$ with the hand
                $$
                begin{aligned}
                &text{Coeff}_{1/z}
                frac {e^{1/z}}{(z-i)^3}
                \
                &=
                text{Coeff}_{1/z}
                -icdot e^{1/z}cdotfrac 1{(1+iz)^3}
                \
                &=text{Coeff}_{1/z}
                -icdot
                left(
                frac 1{0!}
                +frac 1{1!}frac 1z
                +frac 1{2!}frac 1{z^2}
                +frac 1{3!}frac 1{z^3}
                +dotsright)
                \
                &qquadcdot
                left(
                binom 22
                +binom 32 (-iz)
                +binom 42(-iz)^2
                %+binom 52(-iz)^3
                +dots
                right)
                \
                &=
                -ileft(
                frac 1{1!}binom 22
                +frac 1{2!}binom 32 (-i)
                +frac 1{3!}binom 42 (-i)^2
                %+frac 1{4!}binom 52 (-i)^3
                +dots
                right)
                \
                &=dots
                end{aligned}
                $$

                and in the above sum each $frac 1{(n+1)!}binom{n+2}2
                =frac 1{(n+1)!}cdotfrac{(n+2)!}{n!2!}=frac{(n+2)}{n!2!}$
                has to be explained. We split the nummerator in two parts, one with $n$, one with $2$, and will get some parts of the exponential series in $-i$. One of them needs the aid of the missing $1/0!$, so it is acceptable that we get the killing contribution for the first residue.






                share|cite|improve this answer
























                  0












                  0








                  0






                  OK, i will type an answer, after the comment. The result $0$ is correct. We change variables, $w=1/z$. The contour $C$, circle with radius $5$ goes into the circle $C'$ with radius $1/5$. Then we have
                  $$
                  begin{aligned}
                  int_Cfrac {e^{1/z}}{(z-i)^3}; dz
                  &=
                  int_{C'}frac{e^w}{left(frac 1w-iright)^3}; left(-frac 1{w^2}right);dw
                  \
                  &=
                  -int_{C'}frac {w^3e^w}{w^2(1-iw)^3};dw
                  \
                  &=
                  -int_{C'}frac {we^w}{(1-iw)^3};dw
                  .
                  end{aligned}
                  $$

                  There is no pole inside $C'$. (The pole $w=-i$ is outside.)
                  So the integral is zero, Residue Theorem.



                  Numerically, pari/gp:



                  ? intnum( t=0, 2*Pi, 
                  exp( (cos(t)-I*sin(t))/5 )
                  / (5*(cos(t)+I*sin(t)) - I)^3
                  * 5*(-sin(t)+I*cos(t)) )
                  %1 = 1.4190890450537301016459116208947201222 E-21
                  - 1.9571617134379963221261930053722380946 E-21*I


                  (Manually broken lines to fit in the window.)





                  This is of course better than computing the two residues of the initial function (in $0$ and in $i$), which are in $i$...



                  sage: var('z');
                  sage: f = exp(1/z) / (z-i)^3
                  sage: f.residue(z==i)
                  (I + 1/2)*e^(-I)


                  computed with sage, and in $0$ with the hand
                  $$
                  begin{aligned}
                  &text{Coeff}_{1/z}
                  frac {e^{1/z}}{(z-i)^3}
                  \
                  &=
                  text{Coeff}_{1/z}
                  -icdot e^{1/z}cdotfrac 1{(1+iz)^3}
                  \
                  &=text{Coeff}_{1/z}
                  -icdot
                  left(
                  frac 1{0!}
                  +frac 1{1!}frac 1z
                  +frac 1{2!}frac 1{z^2}
                  +frac 1{3!}frac 1{z^3}
                  +dotsright)
                  \
                  &qquadcdot
                  left(
                  binom 22
                  +binom 32 (-iz)
                  +binom 42(-iz)^2
                  %+binom 52(-iz)^3
                  +dots
                  right)
                  \
                  &=
                  -ileft(
                  frac 1{1!}binom 22
                  +frac 1{2!}binom 32 (-i)
                  +frac 1{3!}binom 42 (-i)^2
                  %+frac 1{4!}binom 52 (-i)^3
                  +dots
                  right)
                  \
                  &=dots
                  end{aligned}
                  $$

                  and in the above sum each $frac 1{(n+1)!}binom{n+2}2
                  =frac 1{(n+1)!}cdotfrac{(n+2)!}{n!2!}=frac{(n+2)}{n!2!}$
                  has to be explained. We split the nummerator in two parts, one with $n$, one with $2$, and will get some parts of the exponential series in $-i$. One of them needs the aid of the missing $1/0!$, so it is acceptable that we get the killing contribution for the first residue.






                  share|cite|improve this answer












                  OK, i will type an answer, after the comment. The result $0$ is correct. We change variables, $w=1/z$. The contour $C$, circle with radius $5$ goes into the circle $C'$ with radius $1/5$. Then we have
                  $$
                  begin{aligned}
                  int_Cfrac {e^{1/z}}{(z-i)^3}; dz
                  &=
                  int_{C'}frac{e^w}{left(frac 1w-iright)^3}; left(-frac 1{w^2}right);dw
                  \
                  &=
                  -int_{C'}frac {w^3e^w}{w^2(1-iw)^3};dw
                  \
                  &=
                  -int_{C'}frac {we^w}{(1-iw)^3};dw
                  .
                  end{aligned}
                  $$

                  There is no pole inside $C'$. (The pole $w=-i$ is outside.)
                  So the integral is zero, Residue Theorem.



                  Numerically, pari/gp:



                  ? intnum( t=0, 2*Pi, 
                  exp( (cos(t)-I*sin(t))/5 )
                  / (5*(cos(t)+I*sin(t)) - I)^3
                  * 5*(-sin(t)+I*cos(t)) )
                  %1 = 1.4190890450537301016459116208947201222 E-21
                  - 1.9571617134379963221261930053722380946 E-21*I


                  (Manually broken lines to fit in the window.)





                  This is of course better than computing the two residues of the initial function (in $0$ and in $i$), which are in $i$...



                  sage: var('z');
                  sage: f = exp(1/z) / (z-i)^3
                  sage: f.residue(z==i)
                  (I + 1/2)*e^(-I)


                  computed with sage, and in $0$ with the hand
                  $$
                  begin{aligned}
                  &text{Coeff}_{1/z}
                  frac {e^{1/z}}{(z-i)^3}
                  \
                  &=
                  text{Coeff}_{1/z}
                  -icdot e^{1/z}cdotfrac 1{(1+iz)^3}
                  \
                  &=text{Coeff}_{1/z}
                  -icdot
                  left(
                  frac 1{0!}
                  +frac 1{1!}frac 1z
                  +frac 1{2!}frac 1{z^2}
                  +frac 1{3!}frac 1{z^3}
                  +dotsright)
                  \
                  &qquadcdot
                  left(
                  binom 22
                  +binom 32 (-iz)
                  +binom 42(-iz)^2
                  %+binom 52(-iz)^3
                  +dots
                  right)
                  \
                  &=
                  -ileft(
                  frac 1{1!}binom 22
                  +frac 1{2!}binom 32 (-i)
                  +frac 1{3!}binom 42 (-i)^2
                  %+frac 1{4!}binom 52 (-i)^3
                  +dots
                  right)
                  \
                  &=dots
                  end{aligned}
                  $$

                  and in the above sum each $frac 1{(n+1)!}binom{n+2}2
                  =frac 1{(n+1)!}cdotfrac{(n+2)!}{n!2!}=frac{(n+2)}{n!2!}$
                  has to be explained. We split the nummerator in two parts, one with $n$, one with $2$, and will get some parts of the exponential series in $-i$. One of them needs the aid of the missing $1/0!$, so it is acceptable that we get the killing contribution for the first residue.







                  share|cite|improve this answer












                  share|cite|improve this answer



                  share|cite|improve this answer










                  answered Nov 29 '18 at 19:23









                  dan_fulea

                  6,2151312




                  6,2151312























                      0















                      I thought it might be instructive to present an approach that is equivalent to using the residue at infinity, but perhaps a bit simpler in this case. To that end we proceed.






                      Inasmuch as the singularities of the integrand lie inside the unit circle, Cauchy's Integral Theorem guarantees that for $R>5$



                      $$oint_{|z|=5}frac{e^{1/z}}{(z-i)^3},dz=oint_{|z|=R}frac{e^{1/z}}{(z-i)^3},dz$$



                      Now, we have the simple estimate



                      $$begin{align}
                      left|oint_{|z|=R}frac{e^{1/z}}{(z-i)^3},dzright|&=left|int_0^{2pi} frac{e^{frac1Re^{-iphi}}}{(Re^{iphi}-i)^3},iRe^{iphi},dphiright|\\
                      &le frac{2pi R^2}{(R-1)^4}
                      end{align}$$



                      Finally, letting $Rto infty$, we find



                      $$oint_{|z|=5}frac{e^{1/z}}{(z-i)^3},dz=0$$



                      And we are done!






                      share|cite|improve this answer





















                      • Please let me know how I can improve my answer. I really want to give you the best answer I can. And Happy Holidays! ;-)
                        – Mark Viola
                        Dec 16 '18 at 18:23
















                      0















                      I thought it might be instructive to present an approach that is equivalent to using the residue at infinity, but perhaps a bit simpler in this case. To that end we proceed.






                      Inasmuch as the singularities of the integrand lie inside the unit circle, Cauchy's Integral Theorem guarantees that for $R>5$



                      $$oint_{|z|=5}frac{e^{1/z}}{(z-i)^3},dz=oint_{|z|=R}frac{e^{1/z}}{(z-i)^3},dz$$



                      Now, we have the simple estimate



                      $$begin{align}
                      left|oint_{|z|=R}frac{e^{1/z}}{(z-i)^3},dzright|&=left|int_0^{2pi} frac{e^{frac1Re^{-iphi}}}{(Re^{iphi}-i)^3},iRe^{iphi},dphiright|\\
                      &le frac{2pi R^2}{(R-1)^4}
                      end{align}$$



                      Finally, letting $Rto infty$, we find



                      $$oint_{|z|=5}frac{e^{1/z}}{(z-i)^3},dz=0$$



                      And we are done!






                      share|cite|improve this answer





















                      • Please let me know how I can improve my answer. I really want to give you the best answer I can. And Happy Holidays! ;-)
                        – Mark Viola
                        Dec 16 '18 at 18:23














                      0












                      0








                      0







                      I thought it might be instructive to present an approach that is equivalent to using the residue at infinity, but perhaps a bit simpler in this case. To that end we proceed.






                      Inasmuch as the singularities of the integrand lie inside the unit circle, Cauchy's Integral Theorem guarantees that for $R>5$



                      $$oint_{|z|=5}frac{e^{1/z}}{(z-i)^3},dz=oint_{|z|=R}frac{e^{1/z}}{(z-i)^3},dz$$



                      Now, we have the simple estimate



                      $$begin{align}
                      left|oint_{|z|=R}frac{e^{1/z}}{(z-i)^3},dzright|&=left|int_0^{2pi} frac{e^{frac1Re^{-iphi}}}{(Re^{iphi}-i)^3},iRe^{iphi},dphiright|\\
                      &le frac{2pi R^2}{(R-1)^4}
                      end{align}$$



                      Finally, letting $Rto infty$, we find



                      $$oint_{|z|=5}frac{e^{1/z}}{(z-i)^3},dz=0$$



                      And we are done!






                      share|cite|improve this answer













                      I thought it might be instructive to present an approach that is equivalent to using the residue at infinity, but perhaps a bit simpler in this case. To that end we proceed.






                      Inasmuch as the singularities of the integrand lie inside the unit circle, Cauchy's Integral Theorem guarantees that for $R>5$



                      $$oint_{|z|=5}frac{e^{1/z}}{(z-i)^3},dz=oint_{|z|=R}frac{e^{1/z}}{(z-i)^3},dz$$



                      Now, we have the simple estimate



                      $$begin{align}
                      left|oint_{|z|=R}frac{e^{1/z}}{(z-i)^3},dzright|&=left|int_0^{2pi} frac{e^{frac1Re^{-iphi}}}{(Re^{iphi}-i)^3},iRe^{iphi},dphiright|\\
                      &le frac{2pi R^2}{(R-1)^4}
                      end{align}$$



                      Finally, letting $Rto infty$, we find



                      $$oint_{|z|=5}frac{e^{1/z}}{(z-i)^3},dz=0$$



                      And we are done!







                      share|cite|improve this answer












                      share|cite|improve this answer



                      share|cite|improve this answer










                      answered Nov 30 '18 at 17:45









                      Mark Viola

                      130k1274170




                      130k1274170












                      • Please let me know how I can improve my answer. I really want to give you the best answer I can. And Happy Holidays! ;-)
                        – Mark Viola
                        Dec 16 '18 at 18:23


















                      • Please let me know how I can improve my answer. I really want to give you the best answer I can. And Happy Holidays! ;-)
                        – Mark Viola
                        Dec 16 '18 at 18:23
















                      Please let me know how I can improve my answer. I really want to give you the best answer I can. And Happy Holidays! ;-)
                      – Mark Viola
                      Dec 16 '18 at 18:23




                      Please let me know how I can improve my answer. I really want to give you the best answer I can. And Happy Holidays! ;-)
                      – Mark Viola
                      Dec 16 '18 at 18:23


















                      draft saved

                      draft discarded




















































                      Thanks for contributing an answer to Mathematics Stack Exchange!


                      • Please be sure to answer the question. Provide details and share your research!

                      But avoid



                      • Asking for help, clarification, or responding to other answers.

                      • Making statements based on opinion; back them up with references or personal experience.


                      Use MathJax to format equations. MathJax reference.


                      To learn more, see our tips on writing great answers.





                      Some of your past answers have not been well-received, and you're in danger of being blocked from answering.


                      Please pay close attention to the following guidance:


                      • Please be sure to answer the question. Provide details and share your research!

                      But avoid



                      • Asking for help, clarification, or responding to other answers.

                      • Making statements based on opinion; back them up with references or personal experience.


                      To learn more, see our tips on writing great answers.




                      draft saved


                      draft discarded














                      StackExchange.ready(
                      function () {
                      StackExchange.openid.initPostLogin('.new-post-login', 'https%3a%2f%2fmath.stackexchange.com%2fquestions%2f3018976%2fevaluate-int-c-frace1-zz-i3dz-in-the-circle-z-5%23new-answer', 'question_page');
                      }
                      );

                      Post as a guest















                      Required, but never shown





















































                      Required, but never shown














                      Required, but never shown












                      Required, but never shown







                      Required, but never shown

































                      Required, but never shown














                      Required, but never shown












                      Required, but never shown







                      Required, but never shown







                      Popular posts from this blog

                      How do I know what Microsoft account the skydrive app is syncing to?

                      When does type information flow backwards in C++?

                      Grease: Live!